Multiple Integrals: Find Volume Bounded by Cylinders and Planes

Click For Summary
To find the volume bounded by the cylinders x^2+y^2=1 and the planes y=z, x=0, and z=0 in the first octant, the integration limits for x are from 0 to 1. For each x, y ranges from 0 to the circle defined by y=√(1-x^2), and z ranges from 0 to y. The correct double integral setup is ∫ from 0 to 1 ∫ from 0 to √(1-x^2) y dy dx. The integration process reveals that the square root does not appear until after integrating with respect to y, clarifying the approach to solving the problem. Understanding these limits and the integration order is crucial for accurately calculating the volume.
bodensee9
Messages
166
Reaction score
0

Homework Statement


Hello, I was wondering if someone could help me with the following. Supposed I am asked to find the volume bounded by the cylinders x^2+y^2=1 and the planes y = z, x = 0, z = 0 in the first octant.


Homework Equations


So this is what I tried to do. The boundaries should be: x is between 0 and 1 and y is between the squareroot of (1-x^2) and 0, or you can have y is between 0 and 1 and x is between the squareroot of (1-y^2) and 0. So wouldn't the double integral be the integral of

the squareroot of 1-x^2dydx, where you first evaluate it from 0 to the squareroot of (1-x^2), and then you evaluate it again from 0 to 1? Thanks!


The Attempt at a Solution

 
Physics news on Phys.org
Since you are in the first octant, yes, x runs between 0 and 1. For each x, then y runs from 0 up to the circle, y= \sqrt{1- x^2}. Finally, for each x and y, z runs from 0 up to the plane z= y. The volume is given by
\int_{x=0}^1\int_{y=0}^{\sqrt{1-x^2}}\int_{z=0}^y dzdydx= \int_{x=0}^1\int_{y=0}^{\sqrt{1-x^2}}y dydx[/itex]<br /> No, that is NOT \sqrt{1- x^2}dydx! You don&#039;t get the square root until after integrating with respect to y- and then, since the integral of ydy will involve y<sup>2</sup>, you don&#039;t really have a square root to integrate with respect to x!
 
Oh I see now! Thanks!
 
Question: A clock's minute hand has length 4 and its hour hand has length 3. What is the distance between the tips at the moment when it is increasing most rapidly?(Putnam Exam Question) Answer: Making assumption that both the hands moves at constant angular velocities, the answer is ## \sqrt{7} .## But don't you think this assumption is somewhat doubtful and wrong?

Similar threads

  • · Replies 2 ·
Replies
2
Views
2K
Replies
2
Views
1K
Replies
4
Views
2K
  • · Replies 19 ·
Replies
19
Views
2K
  • · Replies 9 ·
Replies
9
Views
2K
  • · Replies 1 ·
Replies
1
Views
1K
Replies
4
Views
3K
  • · Replies 2 ·
Replies
2
Views
2K
Replies
3
Views
2K
Replies
24
Views
3K